Difference between revisions of "MIE 2016/Day 1/Problem 2"
(Created page with "===Problem 2=== The following system has <math>k</math> integer solutions. We can say that: <math>\begin{cases}\frac{x^2-2x-14}{x}>3\\\\x\leq12\end{cases}</math> (a) <math>...") |
m |
||
(One intermediate revision by the same user not shown) | |||
Line 1: | Line 1: | ||
− | + | ==Problem 2== | |
The following system has <math>k</math> integer solutions. We can say that: | The following system has <math>k</math> integer solutions. We can say that: | ||
Line 16: | Line 16: | ||
− | + | ==Solution== | |
+ | <math>\frac{x^2-2x-14}{x}>3</math> | ||
+ | <math>\frac{x^2-2x-14}{x}-\frac{3x}{x}>0</math> | ||
+ | |||
+ | <math>\frac{x^2-5x-14}{x}>0\begin{cases}x^2-5x-14>0\\x>0\end{cases}</math> | ||
+ | |||
+ | <math>\left.\begin{array}{l}x > 0\\x > 7\\x < -2\end{array}\right\}x\in(-2,0)\cup(7,+\infty)</math> | ||
+ | |||
+ | |||
+ | Adding the other interval we get | ||
+ | |||
+ | |||
+ | <math>x\in(-2,0)\cup(7,12]</math> | ||
+ | |||
+ | |||
+ | If <math>k</math> is the number of integer solutions, then <math>k=7</math>. <math>\boxed{D}</math> | ||
===See Also=== | ===See Also=== |
Latest revision as of 18:29, 8 January 2018
Problem 2
The following system has integer solutions. We can say that:
(a)
(b)
(c)
(d)
(e)
Solution
Adding the other interval we get
If is the number of integer solutions, then .